Sequences and Limits: July 31, 2007 13:35 WSPC /book Trim Size For 9in X 6in Real-Analysis

You might also like

Download as pdf or txt
Download as pdf or txt
You are on page 1of 14

Chapter 1

Sequences and Limits

{ } Let a n be a sequence of real or complex numbers. A necessary and sucient condition for the sequence to converge is that for any > 0 there exists an integer N > 0 such that | a p aq | < holds for all integers p and q greater than N. This is called the Cauchy criterion. Any monotone bounded sequence is convergent. { } For any sequence a n the inferior limit and the superior limit are dened by the limits of monotone sequences { } lim inf a n = lim inf a n , a n+1 , ...
n n

and { } lim sup a n = lim sup a n , a n+1 , ...


n n

respectively. Note that the inferior and superior limits always exist if we adopt as limits. { } A bounded sequence a n converges if and only if the inferior limit coincides with the superior limit.

1
PROBLEMS AND SOLUTIONS IN REAL ANALYSIS World Scientific Publishing Co. Pte. Ltd. http://www.worldscibooks.com/mathematics/6643.html

Problems and Solutions in Real Analysis

Problem 1. 1 Prove that n sin (2n!e) converges to 2 as n .

Problem 1. 2 Prove that the sequence ( )n ( )n ( )n 2 n 1 + + + n n n converges to e/(e 1) as n .

Problem 1. 3 Prove that the sequence ) ( e n/4 n (n+1)/2 11 2 2 n n 1/n converges to 1 as n .


This was proposed by Ces` ro (1888) and solved by P lya (1911). a o

Problem 1. 4 Suppose that a n and bn converge to and as n respectively. Show that the sequence a 0 bn + a1 bn1 + + a n b 0 n converges to as n .

Problem 1. 5

{ } Suppose that a n n0 is a non-negative sequence satisfying a m+n a m + a n + C

for all positive integers m, n and some non-negative constant C. Show that a n /n converges as n .
This is essentially due to Fekete (1923). In various places we encounter this useful lemma in deducing the existence of limits.

PROBLEMS AND SOLUTIONS IN REAL ANALYSIS World Scientific Publishing Co. Pte. Ltd. http://www.worldscibooks.com/mathematics/6643.html

Sequences and Limits

Problem 1. 6

{ } For any positive sequence a n n1 show that ( )n a1 + a n+1 >e an

for innitely many ns, where e is base of the natural logarithm. Prove moreover that the constant e on the right-hand side cannot in general be replaced by any larger number.

Problem 1. 7 For any 0 < < and any positive integer n show the inequality sin + sin 2 sin n + + > 0. 2 n

This was conjectured by Fej r and proved by Jackson (1911) and by Grone wall (1912) independently. Landau (1934) gave a shorter (maybe the shortest) elegant proof. See also Problem 5. 9. Note that

n=1

sin n = n 2

for 0 < < 2, which is shown in Solution 7. 10.

Problem 1. 8 For any real number and any positive integer n show the inequality cos cos 2 cos n 1 + + + . 2 3 n+1 2
This was shown by Rogosinski and Szeg (1928). Verblunsky (1945) gave o another proof. Koumandos (2001) obtained the lower bound 41/96 for n 2. Note that ( ) cos n n + 1 = 2 sin cos log 2 sin 2 n=0 for 0 < < 2. For the simpler cosine sum Young (1912) showed that

n=1

cos n > 1 n

PROBLEMS AND SOLUTIONS IN REAL ANALYSIS World Scientific Publishing Co. Pte. Ltd. http://www.worldscibooks.com/mathematics/6643.html

Problems and Solutions in Real Analysis

for any and positive integer m 2. Brown and Koumandos (1997) improved this by replacing 1 by 5/6.

Problem 1. 9

{ } Given a positive sequence a n n0 satisfying a 1 a 0 + 1 and a n+1


2 an 1 a n1

for any positive integer n, show that a n+1 an converges as n . Show moreover that a n n converges as n , where is the limit of the sequence a n+1 /a n .
This is due to Boyd (1969).

Problem 1. 10 Let E be any bounded closed set in the complex plane containing an innite number of points, and let Mn be the maximum of | V(x1 , ..., x n ) | as the points x1 , ..., x n run through the set E, where V(x1 , ..., x n ) = (x i x j )
1i< jn 2/(n(n1)) is the Vandermonde determinant. Show that Mn converges as n .

This is due to Fekete (1923) and the limit


2/(n(n1)) (E ) = lim Mn n

is called the transnite diameter of E. See Problem 15. 9.

PROBLEMS AND SOLUTIONS IN REAL ANALYSIS World Scientific Publishing Co. Pte. Ltd. http://www.worldscibooks.com/mathematics/6643.html

Sequences and Limits

Solutions for Chapter 1


Solution 1. 1 Let rn and n be the integral and fractional parts of the number n!e respectively. Using the expansion e=1+ we have 1 1 1 + + + + , 1! 2! n!

( ) rn = n! 1 + 1 + 1 + + 1 1! 2! n! 1 = 1 + n + , n + 1 (n + 1)(n + 2) 1 < n+1 1 1 1 + + = . 2 n + 1 (n + 1) n

since
n

<

Thus sin (2n!e) = sin (2 n ). Note that this implies the irrationality of e. Since n n converges to 1 as n , we have
n

lim n sin (2 n ) = lim

sin (2 n )
n

= 2.

Hence n sin (2n!e) converges to 2 as n . Remark. More precisely one gets


n

( ) 1 1 1 3 +O 4 ; n n n

hence we have n sin (2n!e) = 2n ( 5) 43 3 n n +O n n 3 ( ) 2(2 2 3) 1 = 2 + +O 3 2 3n n


n

as n .

PROBLEMS AND SOLUTIONS IN REAL ANALYSIS World Scientific Publishing Co. Pte. Ltd. http://www.worldscibooks.com/mathematics/6643.html

Problems and Solutions in Real Analysis

Solution 1. 2 { } Let d n be any monotone increasing sequence of positive integers diverging to and satisfying d n < n for n > 1. We divide the sum into two parts as follows. ( )n ( )n ( )n a = 1 + 2 + + n 1 dn , n n n n ( ( )n )n bn = n d n + + n . n n First the sum a n is roughly estimated above by 1 nn
0 nd n

( )n (n d n ) n+1 dn x dx = < 1 . (n + 1)n n n


n

Now using the inequality log(1 x) + x < 0 valid for 0 < x < 1 we obtain 0 < a n < e n log(1 dn /n) < e dn , which converges to 0 as n . Next by using Taylors formula for log(1 x) we can take a positive constant c1 such that the inequality | log(1 x) + x | c1 x 2 holds for any | x | 1/2. Thus for any integer n satisfying d n /n 1/2 we get ) ( 2 k c1 k 2 c1 d n +k n log 1 n n n
2 for 0 k d n . Suppose further that d n /n converges to 0 as n . For exam[ 1/3 ] satises all the conditions imposed above. Next take a positive ple d n = n constant c2 satisfying

| e x 1 | c2 | x |
2 for any | x | 1. Since c1 d n /n 1 for all suciently large n, we have ( )n k ek 1 1 = e n log(1k/n)+k 1 n

2 c1 c2 d n . n

PROBLEMS AND SOLUTIONS IN REAL ANALYSIS World Scientific Publishing Co. Pte. Ltd. http://www.worldscibooks.com/mathematics/6643.html

Sequences and Limits

Dividing both sides by e k and summing from k = 0 to d n , we get )n dn ( 2 c1 c2 d n dn k k 1 e k e . n n k=0 k=0 Hence bn which implies bn
dn 2 ec1 c2 d n , (e 1)n

k=0

e k

<

( ) 2 c1 c2 d n e e + e dn . e1 e1 n

Therefore a n + bn converges to e/(e 1) as n . Solution 1. 3 One can easily verify that the function f (x) = x log x satises all the conditions stated in Problem 5.7. Therefore the logarithm of the given sequence converges to f (1) f (0+) = 0, 2 hence the limit is 1. Solution 1. 4 Let M be an upper bound of the two convergent sequences | a n | and | bn |. For any > 0 we can take a positive integer N satisfying | a n | < and | bn | < for all integers n greater than N. If n is greater than N 2 , then | a k bnk | | (a k )bnk + (bnk ) | (M + | | ) ] [ for any integer k in the interval n , n n . Therefore 1 n

k=0

ak bnk

1 n

( ) [ n ] + 1 + 2 | | + M 2 n ( ) n + 1 (M + | | ) + 2 | | + M 2 . n

n kn n

| a k bnk |

PROBLEMS AND SOLUTIONS IN REAL ANALYSIS World Scientific Publishing Co. Pte. Ltd. http://www.worldscibooks.com/mathematics/6643.html

Problems and Solutions in Real Analysis

We can take n so large that the last expression is less than (M + | | + 1) . Solution 1. 5 For an arbitrary xed positive integer k we put n = qk + r with 0 r < k. Since a n = a qk+r q(a k + C) + a r , we have an ak + C ar + . n k n Taking the limit as n , we get lim sup
n

an ak + C . n k

The sequence a n /n is therefore bounded. Since k is arbitrary, we may conclude that an ak lim sup lim inf , n k k n which means the convergence of a n /n. Solution 1. 6 Without loss of generality we may put a 1 = 1. Suppose, contrary to the conclusion, that there is an integer N satisfying )n ( 1 + a n+1 e an for all n N. Put sj,k = exp ( )

1 1 + + j k

for any integers j k. Since 0 < a n+1 e 1/n a n 1, we get successively 0<a n+1 s n,n a n 1, 0 < a n+2 s n,n+1 a n s n+1,n+1 1, . . . 0<a n+k+1 s n,n+k a n s n+1,n+k s n+k,n+k 1 for any non-negative integer k. Hence it follows that an > 1 s n,n + 1 s n,n+1 + + 1 s n,n+k .

PROBLEMS AND SOLUTIONS IN REAL ANALYSIS World Scientific Publishing Co. Pte. Ltd. http://www.worldscibooks.com/mathematics/6643.html

Sequences and Limits

On the other hand, using the inequality ( n+ j ) 1 n1 dx > exp = , s n,n+ j x n+ j n1 we get an >
k

j=0

n1 , n+ j

which is a contradiction, since the right-hand side diverges to as k . To see that the bound e cannot be replaced by any larger number, consider the case a n = n log n for n 2. Then ( )n ( ( ( 1 ))) a 1 + (n + 1) log(n + 1) 1 = exp n log 1 + + O n log n n n log n ( ( 1 )) = exp 1 + O , log n which converges to e as n . Solution 1. 7 Denote by s n () the left-hand side of the inequality to be shown. Write for 2 for brevity. Since ( ) e i + e 2i + + e ni sn () = = cos (n + 1) sin n , sin

we obtain the candidates for extreme points of s n () on the interval (0, ] by solving the equations cos (n + 1) = 0 and sin n = 0, as follows: 2 3 4 , , , , ... n+1 n n+1 n where the last two candidates are (n 1)/(n + 1) and if n is even, and (n 1)/n and n/(n + 1) if n is odd. In any case sn () vanishes at least at n points in the interval (0, ). Since sn () can be expressed as a polynomial in cos of degree n and cos maps the interval [0, ] onto [1, 1] homeomorphically, this polynomial possesses at most n real roots in [1, 1]. Therefore all these roots must be simple and give the actual extreme points of s n () except for = . Clearly s n () is positive in the right neighborhood of the origin, and the maximal and minimal points stand in line alternately from left to right. Thus s n () attains its minimal values at the

PROBLEMS AND SOLUTIONS IN REAL ANALYSIS World Scientific Publishing Co. Pte. Ltd. http://www.worldscibooks.com/mathematics/6643.html

10

Problems and Solutions in Real Analysis

points 2 /n (0, ) when n 3. In the cases n = 1 and n = 2, however, s n () has no minimal points in (0, ). Now we will show that s n () is positive on the interval (0, ) by induction on n. This is clear for n = 1 and n = 2 since s 1 () = sin and s 2 () = (1+cos ) sin . Suppose that s n1 () > 0 for some n 3. Then the minimal values of s n () are certainly attained at some points 2 /n in (0, ), whose values are ( ) ( ) 2 2 sin 2 sn = s n1 + n n n ) ( 2 > 0. = s n1 n Therefore s n () > 0 on the interval (0, ). Remark. Landau (1934) gave the following elegant shorter proof using mathematical induction on n. Suppose that s n1 () > 0 on (0, ). If s n attains the non-positive minimum at some point, say , then sn ( ) = 0 implies ( ) 1 sin n + = sin 2 2 and hence ( ) 1 cos n + = cos . 2 2 ) ( ) 1 1 sin n = sin n + cos cos n + sin 2 2 2 2 = sin cos cos sin , 2 2 2 2

Since

being equal either to 0 or sin 0 according to the sign. We are led to a contradiction. Solution 1. 8 The proof is substantially based on Verblunsky (1945). Write for 2 for brevity. Let cn () be the left-hand side of the inequality to be shown. It suces to conne ourselves to the interval [0, /2]. Clearly c1 () = cos /2 1/2 and c 2 () = 2 1 1 41 cos2 + cos , 3 2 3 96

PROBLEMS AND SOLUTIONS IN REAL ANALYSIS World Scientific Publishing Co. Pte. Ltd. http://www.worldscibooks.com/mathematics/6643.html

Sequences and Limits

11

and we assume that n 3. Note that cos n = sin (2n + 1) sin (2n 1) 2 sin sin2 (n + 1) 2 sin2 n + sin2 (n 1) = , 2 sin2

{ } whose numerator is the second dierence of the positive sequence sin 2 n . Using this formula we get cn () =
n sin2 (k + 1) 2 sin2 k + sin2 (k 1) 1 , k+1 2 sin2 k=1

which can be written as ( 1 2 sin2 sin2 2 2 sin2 (n 1) + + + 3 12 n(n 2 1) 2 sin2 Hence we obtain cn ()

) (n 1) sin2 n sin2 (n + 1) + . n(n + 1) n+1

1 cos2 sin2 (n + 1) sin2 n + + 3 6 2(n + 1) sin2 1 sin2 sin (2n + 1) = + . 6 6 2(n + 1) sin

For any satisfying sin (2n + 1) 0 we obviously have cn () 1/3. Moreover if belongs to the interval (3/(2n + 1), /2), then using Jordans inequality sin 2/, cn () 1 1 3 2(n + 1) sin (3/(2n + 1)) 1 2n + 1 1 > . 3 12(n + 1) 2

Thus it suces to consider the interval [/(2n + 1), 2/(2n + 1)]. In general, we consider an interval of the form [ ] , . 2n + 1 2n + 1 For any satisfying sin (2n + 1) c on this interval it follows that cn () 1 sin2 c . 6 6 2(n + 1) sin

PROBLEMS AND SOLUTIONS IN REAL ANALYSIS World Scientific Publishing Co. Pte. Ltd. http://www.worldscibooks.com/mathematics/6643.html

12

Problems and Solutions in Real Analysis

Now the right-hand side can be written as 1/6(sin ), where (x) is a concave function; hence, the maximum of is attained at an end point of that interval. By using sin 7 sin we get ( sin ) 1 c = sin2 + 2n + 1 6 2n + 1 2(n + 1) sin (/(2n + 1)) () 2 c 2n + 1 + . 2 2(n + 1) 7 sin (/7) 6(2n + 1) , 7

Since n 3, the last expression is less than c () 2 + . 294 7 sin (/7) Similarly we get an estimate for another end point. For = 1 and = 4/3 we can take c = 3/2 so that the value of at the corresponding end point is less than 0.319 and 0.28 respectively. Similarly for = 4/3 and = 2 we can take c = 1 so that the value of is less than 0.314 and 0.318 respectively. Therefore the maximum of on the interval [/(2n+1), 2/(2n+1)] is less than 1/3, which implies that cn () > 1/2. Solution 1. 9 We rst show that a n+1 1 >1+ an a0 (1. 1)

by induction on n. When n = 0 this holds by the assumption. Put = 1 + 1/ a 0 for brevity. Suppose that (1. 1) holds for n m. We then have a k > k a 0 for 1 k m + 1. Thus
m+1 m+1 a k+1 1 a m+2 a 1 ak , a m+1 a 0 ak a k1 ak k=1 k=1

which is less than 1 a0


m+1 k=1

k < a0 ( 1) =

1 . a0

PROBLEMS AND SOLUTIONS IN REAL ANALYSIS World Scientific Publishing Co. Pte. Ltd. http://www.worldscibooks.com/mathematics/6643.html

Sequences and Limits

13

Therefore a m+2 a 1 1 1 > >1+ ; a m+1 a 0 a0 a0 thus (1. 1) holds also for n = m + 1. Let p > q be any positive integers. In the same way,
p p a p+1 a q+1 a k+1 1 ak , ap aq ak a k1 a k=q+1 k=q+1 k

which is less than

pq a0 1 1 < aq . a q k=1 k { } This means that the sequence a n+1 /a n satises the Cauchy criterion since a q diverges to as q . Letting p in the above inequalities, we get a0 a q+1 . aq aq Multiplying both sides by a q / q+1 , we have a0 a q+1 a q q q+1 , q+1 { n} which shows that the sequence a n / also satises the Cauchy criterion. Solution 1. 10 Let 1 , ..., n+1 be the points at which | V(x 1 , ..., x n+1 ) | attains its maximum M n+1 . Since V( 1 , ..., n+1 ) = ( 1 n+1 ) ( n n+1 ), V( 1 , ..., n ) we have M n+1 | 1 n+1 | | n n+1 |. Mn Applying the same argument to each point 1 , ..., n , we get n + 1 similar inequalities whose product gives )n+1 ( M n+1 2 | i j | = M n+1 . Mn i j

PROBLEMS AND SOLUTIONS IN REAL ANALYSIS World Scientific Publishing Co. Pte. Ltd. http://www.worldscibooks.com/mathematics/6643.html

14

Problems and Solutions in Real Analysis

2/(n(n1)) Hence the sequence M n is monotone decreasing.

PROBLEMS AND SOLUTIONS IN REAL ANALYSIS World Scientific Publishing Co. Pte. Ltd. http://www.worldscibooks.com/mathematics/6643.html

You might also like